[ 3 / biz / cgl / ck / diy / fa / ic / jp / lit / sci / vr / vt ] [ index / top / reports ] [ become a patron ] [ status ]
2023-11: Warosu is now out of extended maintenance.

/sci/ - Science & Math


View post   

File: 1.13 MB, 1800x1108, day_46.png [View same] [iqdb] [saucenao] [google]
10569073 No.10569073 [Reply] [Original]

[math]\boxed{
\text{Let }f\text{ be a real function with a continuous third derivative such that}
\\
f(x), f'(x), f''(x), f'''(x)\text{ are positive for all }x\text{. Suppose that }f'''(x)\leq f(x)
\\
\text{for all }x\text{. Show that }f'(x)<2f(x)\text{ for all }x\text{.}
}[/math]

>> No.10569076

Previous Thread >>10566116

>> No.10569166

bro please can we get the prismrivers

>> No.10569366 [DELETED] 

Just glancing at this, do we use Poincare inequality?

>> No.10569516

>>10569073
Thinly veiled intro analysis thread

>> No.10569520

>>10569516
*analysis homework thread

>> No.10569580

This one was tricky. It's a suboptimal inequality, so you have to add in terms oh the RHS to get what you want. I had to start from the beginning multiple times, as well, then fit the suboptimal inequalities together.

>> No.10571107

How much additional math past calc.3 do I need to learn before I can comprehend the answer to this problem?

>> No.10571270

>>10571107
I solved it (I think) with basic calc rules after playing around for a while this morning. So I'd imagine it's a lower level question, like a 1 or 2.